MỘT SỐ BÀI TOÁN SỬ DỤNG HÀNG ĐIỂM ĐIỀU HÒA HAY SƯU ...

Bài toán (ELMO Shortlist 2012)

Cho tam giác ABC và tâm nội tiếp (I)D là chân vuông góc của I xuống BCP là chân vuông góc của I xuống AD. Chứng minh \angle BPD = \angle CPD.

Gọi (c) là đường tròn đường kính ID. Dễ dàng thấy rằng BC,IP,EF theo thứ tự là trục đẳng phương của các cặp đường tròn (c) và (I)(c) và (AFPIE)(AFPIE) và (I). Như vậy BC,IP,EF đồng quy tại J.

Từ đó dễ thấy (JD,BC)=-1 mà PI vuông góc PD nên theo định lý về chùm điều hòa ta được \angle BPD = \angle CPD

Bài toán : Cho tứ giác ABCD nội tiếp (O) có AD,BC giao nhau tại E và AC,BD giao nhau tại P. Gọi M,N là trung điểm của AC,BD. Gọi I là tâm ngoại tiếp tam giác EMN. Chứng minh OP song song EI.

Lời giải :

fffff

Gọi S là trung điểm của EFX theo thứ tự là giao của EF với BD,AC.

Dễ thấy (AC,YP)=-1 mà M là trung điểm của AC nên theo hệ thức Macraulin :

PM.PY=PA.PC

Hoàn toàn tương tự thì PN.PX=PD.PB. Hơn nữa lại có PA.PC=PB.PD=P_{P/(O)} nên có PN.PX=PM.PY

Suy ra M,N,X,Y đồng viên.

Mà dễ dàng thấy S,M,N thẳng hàng theo định lí về đường thẳng Gauss.

Do vậy ta được SM.SN=SX.SY. Cũng dễ thấy (EF,XY)=-1 và S là trung điểm của EF nên theo hệ thức Newton :

SE^2=SX.SY

Ta thu được SE^2=SM.SN. Suy ra SE là tiếp tuyến của (EMN). Từ đó IE vuông góc EF.

Lại theo định lý Brocard thì O là trực tâm tam giác PEF, kéo theo OP vuông góc EF. Như vậy IEsong song OP.

Bài toán : Cho tam giác ABCI_A,I_B,I_C lần lượt là tâm các đường tròn bàng tiếp góc A,B,C(I_A)tiếp xúc BC,CA,AB tại A_1,B_1,C_1(I_B) tiếp xúc BC,CA,AB tại A_2,B_2,C_2(I_C) tiếp xúc BC,CA,AB tại A_3,B_3,C_3. Gọi D,E,F theo thứ tự là giao của các cặp đường thẳng (I_AA_1,B_1C_1),(I_BA_2,B_2C_2),(I_CA_3,B_3C_3). Chứng minh AD,BE,CF đồng quy.

Lời giải :

dfdarlemma

Qua A kẻ đường thẳng d song song với BCI_AA_1 cắt d tại TB_1C_1 cắt d tại U.

Dễ thấy các điểm A,T,B_1,C_1,I_A cùng thuộc đường tròn đường kính AI_A, suy ra :

\angle C_1TI_A=\angle C_1AI_A=\dfrac{\angle A}{2}=\angle B_1AI_A=\angle B_1TI_A

Tức TI_A là phân giác góc B_1TC_1, hơn nữa TI_A vuông góc TU. Do đó T(B_1C_1,UI_A)=-1.

Kéo theo (B_1C_1,UD)=-1. Lại kéo theo A(CB,UD)=-1 mà ta có AU song song BC nên theo định lý về chùm điều hòa, ta được AD đi qua trung điểm của BC hay AD là trung tuyến của tam giác ABC.

Một cách tương tự BE,CF là trung tuyến của tam giác ABC. Như vậy AD,BE,CF đồng quy.

GEO`

Bài toán (Đề thi Olympic Duyên Hải Bắc Bộ 2012-2013 môn toán lớp 11)

Cho tam giác ABC ngoại tiếp (I) và nội tiếp (O). Tiếp điểm với (I) trên BC,CA,AB theo thứ tự là D,E,FH là chân vuông góc hạ từ D xuống EFAH cắt (O) tại G. Tiếp tuyến tại G của (O) cắt BC tại T. Chứng minh tam giác TDG cân.

Lời giải :

OLPDHB2B

Theo định lí sin trong tam giác BGC :

\dfrac{BG}{GC}=\dfrac{sin\angle GBC}{sin\angle GCB}=\dfrac{sin\angle HAE}{sin\angle HAF}

Lại theo định lí sin trong các tam giác AFH,AEH :

\dfrac{HF}{sin\angle HAF}=\dfrac{HA}{sin\angle AFH}=\dfrac{HA}{sin\angle AEH}=\dfrac{HE}{sin\angle HAE}\Rightarrow \dfrac{sin\angle HAF}{sin\angle HAE}=\dfrac{HF}{HE}

Lại có :

\dfrac{HF}{HE}=\dfrac{FD.cos\angle EFD}{DE.cos\angle FED}=\dfrac{\dfrac{BD.sin\angle B}{sin\angle BDF}.cos\left ( \pi -\angle AFE-\angle BFD \right )}{\dfrac{DC.sin\angle C}{sin\angle EDC}.cos\left ( \pi -\angle DEC-\angle FEA \right )}=\dfrac{BD}{DC}

Từ đó ta có :

\dfrac{BG}{GC}=\dfrac{BD}{DC}

Suy ra GD là phân giác trong tam giác BGC.

Kẻ phân giác ngoài GS ta có hàng điều hòa phân giác :

(SDBC)=-1

Do đó nếu ta gọi trung điểm của SD là T' thì theo hệ thức Maclaurin :

\overline{BT'}.\overline{BD}=\overline{BC}.\overline{BS}

\overline{CT'}.\overline{CD}=\overline{CB}.\overline{CS}

Lần lượt chia hai đẳng thức trên theo vế :

\dfrac{\overline{BT'}}{\overline{CT'}}=-\dfrac{\overline{BS}}{\overline{CS}}.\dfrac{\overline{BD}}{\overline{CD}}

Chú ý theo tính chất phân giác trong và phân giác ngoài ta có :

\dfrac{BS}{CS}=\dfrac{BG}{GC}=\dfrac{BD}{DC}

Do đó mà \dfrac{\overline{BT'}}{\overline{CT'}}=\dfrac{BG^2}{CG^2}

Hơn nữa vì hai tam giác TGB và TCG đồng dạng nên ta có :

\dfrac{BG^2}{CG^2}=\dfrac{TB^2}{TG^2}=\dfrac{TB^2}{\overline{TC}.\overline{TB}}=\dfrac{\overline{BT}}{\overline{CT}}

Suy ra :

\dfrac{\overline{BT}}{\overline{CT}}=\dfrac{\overline{BT'}}{\overline{CT'}}\Rightarrow T\equiv T'

Từ đó để ý rằng T chính là tâm ngoại tiếp tam giác SGD nên TG=TD. Vậy tam giác TGD cân.

Bài toán (Kiểm tra đội tuyển lớp 10 THPT Chuyên Lương Thế Vinh, Đồng Nai 2012-2013).

Cho đường tròn (I) nội tiếp tam giác ABC và tiếp xúc BC,CA,AB lần lượt tại D,E,F. Đường thẳng qua A song song với BC cắt EF tại K. Gọi M là trung điểm của BC. Chứng minh rằng : IMvuông góc với DK.

Lời giải :

Bổ đề : Cho tam giác ABC ngoại tiếp (I), các tiếp điểm trên BC,CA,AB lần lượt là D,E,FM là trung điểm của BC. Khi đó ta có ID,EF,AM đồng quy.

Chứng minh tại đây

Quay trở lại bài toán :

untitled

Theo bổ đề ta có AM,EF,ID đồng quy tại G.

Ta có AK\parallel BC,BM=CM nên A(BCMK)=-1.

Kéo theo A(EFKG)=D(EFKG)=-1

Qua I kẻ tia Ix song song với BC. Tương tự trên ta có (Ix,IM,IC,IB)=-1

Suy ra (Ix,IM,IC,IB)=(DG,DK,DE,DF)=-1. Kết hợp với DI\perp Ix,DF\perp IB,DE\perp IC ta suy ra DK\perp IM

Bài toán : (Gặp gỡ Toán học lần IV) Cho điểm P nằm ngoài đường tròn (O)PC là tiếp tuyến của (O)kẻ từ PPAB là cát tuyến. CD là đường kính của (O). Gọi E là giao điểm của PO với BD. Chứng minh rằng CE vuông góc với CA.

Lời giải :

fsaeawq

Kẻ tiếp tuyến PG đến (O) (G là tiếp điểm)

Gọi I là giao điểm của PO với AD. Dễ thấy DG\parallel IE (cùng vuông góc với GC)

Dễ thấy ACBG là tứ giác điều hòa nên D\left ( BACG \right )=-1 mà đường thẳng PO cắt DA,DB,DClần lượt tại I,E,O và DG\parallel IE nên O là trung điểm của IE.

Kết hợp với O là trung điểm của CD ta có CIDE là hình bình hành.

Suy ra CE\parallel AD mà CA\perp AD nên CE\perp AC (điều phải chứng minh)

Bài toán : Cho tứ giác ABCD nội tiếp. E,F lần lượt là giao điểm của các cặp (AB,CD),(AD,BC)M,N lần lượt là trung điểm của AC,BD. Gọi P,Q lần lượt là giao của AC,BD với EF. Chứng minh rằng M,N,P,Q đồng viên.

Lời giải :

untitled

Gọi I là giao điểm của AC,BD. Gọi L là giao điểm của FI với DC.

Ta có (ELDC)=-1 (hàng điều hòa tứ giác toàn phần)

Qua phép chiếu xuyên tâm F ta được (PIAC)=-1. Vì M là trung điểm của AC nên theo hệ thức Maclaurin :

\overline{IP}.\overline{IM}=\overline{IA}.\overline{IC}

Hoàn toàn tương tự : \overline{IQ}.\overline{IN}=\overline{IB}.\overline{ID}

Mà \overline{IA}.\overline{IC}=P_{I/{ABCD}}=\overline{IB}.\overline{ID}

Dẫn đến \overline{IN}.\overline{IQ}=\overline{IP}.\overline{IM}

Điều này chứng tỏ M,N,P,Q đồng viên.

Bài toán : Cho tứ giác ABCD nội tiếp. M,N lần lượt là trung điểm của AB,CD. Đường tròn ngoại tiếp tam giác ABN cắt CD tại P. Đường tròn ngoại tiếp tam giác CDM cắt AB tại Q. Chứng minh rằng AC,BD,PQ đồng quy.

Lời giải :

d

Khi AB\parallel CD thì kết quả là hiển nhiên. Xét AB,CD không song song. Gọi W=AB\cap CD.

Ta có A,B,C,D đồng viên nên \overline{WA}.\overline{WB}=\overline{WD}.\overline{WC}

Ta có A,B,P,N đồng viên nên \overline{WA}.\overline{WB}=\overline{WP}.\overline{WN}

Suy ra \overline{WP}.\overline{WN}=\overline{WC}.\overline{WD} mà N là trung điểm của CD nên theo hệ thức Maclaurin ta có (WPCD)=-1. Tương tự (WQAB)=-1

Suy ra \left ( WPCD \right )=\left ( WQAB \right ), như vậy PQ,AC,BD đồng quy.

Bài toán (IMO Shortlist 1994) Cho tam giác ABC có D,E,F lần lượt là tiếp điểm trên BC,CA,ABcủa đường tròn nội tiếp tam giác. Gọi X là một điểm bên trong tam giác ABC sao cho đường tròn nội tiếp tam giác XBC tiếp xúc với BC tại D, tiếp xúc với XB,XC theo thứ tự tại Y,Z. Chứng minh E,F,Y,Z đồng viên.

Lời giải :

untitled

Gọi J,J' lần lượt là giao điểm của EF,YZ với BC. Ta chứng minh J\equiv J'.

Dễ thấy AD,BE,CF đồng quy (tại điểm Gergonne của tam giác ABC) nên (JDBC)=-1 (hàng điều hòa tứ giác toàn phần)

Tương tự (J'DBC)=-1, suy ra (JDBC)=(J'DBC). Suy ra J\equiv J'.

Từ đó, JD là tiếp tuyến chung của hai đường tròn nội tiếp tam giác ABC,XBC nên : JE.JF=JD^2=JY.JZ

Suy ra các điểm E,F,Y,Z đồng viên.

Bài toán : Cho đường tròn nội tiếp (O) của tam giác ABC. Gọi M là trung điểm của BCAM cắt (O) tại hai điểm K,L (K nằm giữa A,L). Qua K kẻ đường thẳng song song với BC cắt (O) tại điểm thứ hai là X. Qua L, kẻ đường thẳng song song với BC cắt (O) tại điểm thứ hai là YAX,AY cắt BC tại Q,P. Chứng minh M là trung điểm của PQ.

Lời giải :

Bổ đề : Cho tam giác ABC ngoại tiếp (O), tiếp điểm của (O) trên BC,CA,AB lần lượt là D,E,F. Gọi M là trung điểm của BC. Chứng minh AM,EF,OD đồng quy.

Chứng minh bổ đề :

fsdfsdfsdatyu

Gọi I là giao của OD với EF. Ta chứng minh AI đi qua trung điểm M của BC. Ta sẽ xây dựng nên chùm điều hòa.

Qua A kẻ đường thẳng song song với BC cắt OD tại J, cắt EF tại S.

Ta có \widehat{EJI}=\widehat{EAO}=\widehat{OAF}=\widehat{FJI} (các điểm A,F,O,E,J đồng viên)

Tức là JI là phân giác góc FJE. Mặt khác JI \perp JS \left ( JI \perp BC,BC\parallel JS \right )

Do đó chùm J(FEIS)=-1, tức A\left ( BCIS \right )=-1. Mặt khác chùm A(BCIS) có BC\parallel AS nên AI đi qua trung điểm M của BC. Bổ đề được chứng minh.

Quay trở lại bài toán :

HĐĐH

Gọi R là giao của YL với AQ

Theo bổ đề trên ta có AM,OD,EF đồng quy tại W hay KL,OD,EF đồng quy tại W. Mà XKYL là hình thang cân có OD là trục đối xứng, lại có OD cắt KL ở W nên W cũng thuộc XY.

Ta có (AWKL)=-1 (hàng điều hòa về đường tròn) nên X(AWKL)=X(RYKL)=-1, mà XK\parallel RY nên L là trung điểm của RY, tức YL=LR

Theo định lí Thales \dfrac{YL}{PM}=\dfrac{AL}{AM}=\dfrac{LR}{MQ}\Rightarrow MP=MQ, tức M là trung điểm của PQ.

Ta có điều phải chứng minh.

Bài toán : Cho tam giác ABC ngoại tiếp đường tròn (I)D,E,F lần lượt là tiếp điểm của (I) với BC,CA,ABAD cắt (I) tại XBX,CX theo thứ tự cắt (I) tại Y,ZAY,AZ lần lượt cắt I tại R,S. Chứng minh rằng AD,ES,FR đồng quy.

Lời giải :

g

Gọi K là giao điểm của ES với AD. Ta có (AXKD)=S(AXKD)=S(ZXED)

Mặt khác ta thấy ZXED là một tứ giác điều hòa và S là điểm thuộc đường tròn ngoại tiếp tứ giác ZXED, suy ra S(ZXED)=-1\Rightarrow (AXKD)=-1

Tương tự, nếu gọi K' là giao điểm của FR với AD thì \left ( AXK'D \right )=-1.

Như vậy \left ( AXK'D \right )=(AXKD)\Rightarrow K\equiv K'. Hay AD,ES,FR đồng quy.

Bài toán : Cho tam giác ABC ngoại tiếp (I)D là điểm tiếp xúc của (I) với BC. Gọi M là một điểm thuộc đoạn AD. Đường thẳng BM,CM theo thứ tự cắt (I) tại B_1,B_2;C_1,C_2 sao cho BB_1<BB_2,CC_1<CC_2. Chứng minh rằng BC,B_1C_1,B_2C_2 đồng quy.

Lời giải :

untitled

Gọi E,F là hai tiếp điểm với (I) của CA,AB.

Gọi L là giao điểm của EF,BC. Gọi W là giao điểm của AD với (I). Dễ thấy WFDE là một tứ giác điều hòa nên LW là tiếp tuyến của (I). Gọi giao điểm của đường thẳng LB_1 với AD,(I) lần lượt là X,C_0.

Ta có (LXB_1C_0)=-1 (hàng điều hòa về đường tròn) \Rightarrow M(LDBC_0)=-1\;\;\;(1)

Mặt khác dễ thấy AD,BE,CF đồng quy tại điểm Gergonne của tam giác ABC nên (LDBC)=-1 (hàng điều hòa tứ giác toàn phần), do đó \Rightarrow M(LDBC_1)=-1\;\;\;(1)

Từ (1)(2) suy ra M(LDBC_0)=M(LDBC_1)\Rightarrow MC_0\equiv MC_1

Mà C_0\in \left ( I \right ),C_1\in \left ( I \right )\Rightarrow C_0\equiv C_1, tức là B_1C_1 đi qua L.

Tương tự B_2C_2 đi qua L

Kết luận : BC,B_1C_1,B_2C_2 đồng quy.

Bài toán : Trên cạnh BC của tam giác ABC, lấy một điểm N thỏa mãn đồng thời BN=2NC và \widehat{NAB}=\widehat{NAC}. Gọi L là chân đường vuông góc hạ từ B xuống AK. Gọi M là trung điểm của BCvà H là giao điểm của tia ML với AC.

a) Chứng minh FH \perp AC

b) Chứng minh rằng HM là phân giác góc BHN.

Lời giải :

untitled

Bổ đề 1 : Cho hai điểm A,B phân biệt và một số thực k. Khi đó tồn tại duy nhất một điểm H thuộc đường AB và thỏa mãn HA^{2}-HB^{2}=k

Bổ đề 2 : Cho hai đường thẳng a,b. Các điểm A,B lần lượt thuộc a và C,D lần lượt thuộc b. Khi đó a \perp b\Leftrightarrow AC^2-BC^2=AD^2-BD^2

Xem chứng minh hai bổ đề tại đây.

Trở lại bài toán :

a) Đặt AB=c,CA=b,BC=a,\widehat{NAC}=\alpha thì AL=AB.cos\widehat{NAB}=c.cos2\alpha

Theo định lí hàm số cos trong tam giác ALC :

LC^2=AC^2+LA^2-2AC.LA.cos\widehat{NAC}\Leftrightarrow LC^2-LA^2=AC^2-2AC.LA.cos\widehat{NAC}=b^2-2bc.cos\alpha .cos2\alpha

Ta có MC^2-MA^2=\dfrac{a^{2}}{4}-\dfrac{1}{4}\left ( 2b^2+2c^2-a^{2} \right )=\dfrac{a^2-b^2-c^2}{2}=-bc.cosA=-bc.cos \alpha

Như vậy theo bổ đề 2, ta có MH \perp AC\Leftrightarrow ML \perp AC\Leftrightarrow LC^2-LA^2=MC^2-MA^2\Leftrightarrow b^2-2bc.cos\alpha .sin2\alpha =-bc.cos3\alpha \Leftrightarrow b^2+bc.cos3\alpha =2bc.cos\alpha .sin2\alpha \Leftrightarrow \dfrac{b}{c}+cos3\alpha =2cos\alpha .sin2\alpha \;\;\;(*)

Mặt khác theo định lí hàm số sin trong các tam giác NAB,NAC :

\dfrac{BN}{sin\widehat{NAB}}=\dfrac{AB}{sin\widehat{ANB}}\Rightarrow sin\widehat{ANB}=\dfrac{AB.sin\widehat{NAB}}{BN}=\dfrac{3c.sin2\alpha }{2a}

và \dfrac{NC}{sin\widehat{NAC}}=\dfrac{AC}{sin\widehat{ANC}}\Rightarrow sin\widehat{ANC}=\dfrac{AC.sin\widehat{NAC}}{NC}=\dfrac{3b.sin\alpha }{a}

Chú ý rằng vì \widehat{ANB}+\widehat{ANC}=\pi \Rightarrow sin\widehat{ANB}=sin\widehat{ANC}\Rightarrow \dfrac{3c.sin2\alpha }{2a}=\dfrac{3b.sin\alpha }{a}\Rightarrow c=\dfrac{2bsin\alpha }{sin2\alpha }=\dfrac{b}{cos\alpha }

Thay \dfrac{b}{c}=cos\alpha vào (*) thì ta được :

\left ( * \right )\Leftrightarrow cos\alpha +cos3a=2cos\alpha .cos2\alpha. Hiển nhiên đúng.

Như vậy ta có MH \perp AC.

b) Ta có \dfrac{CB}{CN}=3 và 4MB=2BC=3NB=3(MN+MB)\Rightarrow \dfrac{MB}{MN}=3

Suy ra \dfrac{\overline{MB}}{\overline{MN}}=-\dfrac{\overline{CB}}{\overline{CN}}\Rightarrow (BNMC)=-1

Như vậy ta có H\left ( BNMC \right )=-1 mà HM \perp HC (câu a) nên theo định lí về chùm điều hòa ta có HM là phân giác góc BHN.

Bài toán : Cho tam giác ABC ngoại tiếp đường tròn (O). Các tiếp điểm với (O) trên BC,CA,AB lần lượt là D,E,F. Gọi S là giao điểm của EF với BC. Gọi I,J lần lượt là giao điểm của đường thẳng SO với (O). Chứng minh rằng BI,CJ,AD đồng quy.

Lời giải :

untitled

Gọi X là giao điểm của AD với (O) và Y là giao điểm của SO với AD.

Dễ dàng thấy rằng XFDE là một tứ giác điều hòa, từ đó dễ dàng thấy ngay SX là tiếp tuyến tại Xcủa (O).

Ta thấy SX,SD tiếp xúc với (O) tại X,D nên (SYIJ)=-1 (hàng điều hòa về đường tròn)

Mặt khác thì AD,BE,CF đồng quy tại điểm Gergonne của tam giác ABC từ đó có (SDBC)=-1 (hàng điều hòa tứ giác toàn phần)

Suy ra (SYIJ)=(SDBC). Vậy nên YD,BI,CJ đồng quy hay AD,BI,CJ đồng quy.

Bài toán : Cho tam giác ABC và điểm O nằm trong tam giác. Các tia BO,CO lần lượt cắt AC,ABtại E,F. Gọi I là giao điểm của AO,EF. Gọi H là hình chiếu của I trên BC. Chứng minh rằng \widehat{AHE}=\widehat{OHF}

Lời giải :

ser

Gọi J là giao điểm của tia AO với BCS là giao điểm của EF với BC

Ta có \left ( BCJS \right )=-1 (hàng điều hòa tứ giác toàn phần)

\Rightarrow H \left ( EFIS \right )=-1 mà HI \perp HS nên theo định lí về chùm điều hòa ta có HI là phân giác của góc EHF

Tức là \widehat{EHI}=\widehat{FHI}

Cũng vì \left ( BCJS \right )=-1 ta có \left ( FB,FC,FJ,FS \right )=-1\Rightarrow \left ( FO,FA,FJ,FI\right )=-1\Rightarrow H(OAJI)=-1 mà HI \perp HS nên theo định lí về chùm điều hòa ta có HI là phân giác của góc AHO.

Tức là \widehat{AHI}=\widehat{OHI}

Do đó \widehat{AHE}=\widehat{EHI}-\widehat{AHI}=\widehat{FHI}-\widehat{OHI}=\widehat{OHF}

Đây là điều phải chứng minh

Bài toán (China TST 2002) Cho tứ giác lồi ABCD, gọi E,F,P lần lượt là giao điểm của AD và BCAB và CDAC và BD. Gọi O là chân đường vuông góc hạ từ P xuống EF. Chứng minh rằng \widehat{AOD}=\widehat{BOC}

Lời giải :

ChinaTST2002

Gọi I là giao điểm của BD và EF và J là giao điểm của EP với CD.

Ta có \left ( DCJF \right )=-1 (hàng điều hòa tứ giác toàn phần) nên E\left ( DCJF \right )=-1\Rightarrow E\left ( DBPI \right )=-1\Rightarrow O\left ( DBPI \right )=-1

Mà OP \perp OI nên theo định lí về chùm điều hòa, ta có OP là phân giác \widehat{DOC}\Rightarrow \widehat{DOP}=\widehat{BOP}

Hoàn toàn tương tự ta có \widehat{AOP}=\widehat{COP}

Từ đó \widehat{AOD}=\widehat{AOP}-\widehat{DOP}=\widehat{COP}-\widehat{BOP}=\widehat{BOC}

Đây là điều phải chứng minh.

Bài toán : Cho tứ giác ABCD ngoại tiếp đường tròn tâm O. Gọi E,F lần lượt là giao điểm của ACvới (O). Hạ OH vuông góc với BD. Chứng minh rằng \widehat{AHE}=\widehat{CHF}

Lời giải :

Bổ đề : Cho tứ giác ABCD ngoại tiếp đường tròn (O) với các tiếp điểm trên AB,BC,CD,DA lần lượt là M,N,P,Q. Gọi K là giao của MN,PQ và I là giao của AC,BD. Khi đó ta có KO \perp BDK,A,C thẳng hàng, từ đó có (KIAC)=-1

Chứng minh bổ đề : 

fsdfsdfs

\bullet Kẻ hai tiếp tuyến KE,KF với (O). Khi đó tứ giác MENF điều hòa nên EF và các tiếp tuyến tại M,N của (O) đồng quy.Tức là MB,NN,EF đồng quy hay E,F,B thẳng hàng. Tương tự ta có E,F,D thẳng hàng. Suy ra E,F,B,D thẳng hàng.

Mà dễ thấy OK\perp EF\Rightarrow OK\perp BD\;\;\;(1)

\bullet Gọi K' là giao điểm của MN,AC. Khi đó ta có \dfrac{K'A}{K'C}.\dfrac{PC}{PD}.\dfrac{QD}{QA}=\dfrac{K'A}{K'C}.\dfrac{PC}{QA}=\dfrac{K'A}{K'C}.\dfrac{NC}{MA}=\dfrac{K'A}{K'C}.\dfrac{MB}{MA}.\dfrac{NC}{NB}=1

(Chú ý theo tính chất tiếp tuyến mà một số đoạn thẳng bằng nhau AQ=MA,MB=NB,CN=CP,DP=DQ và áp dụng định lí Menelaus cho tam giác ABCvới sự thẳng hàng của K',M,N)

Do đó theo định lí Menelaus cho tam giác ADC ta có P,Q,K' thẳng hàng. Suy ra K\equiv K'

Điều này chứng tỏ A,K,C thẳng hàng  (2)

\bullet Bằng định lí Pascal ta dễ dàng chứng minh được MP,NQ,BD,AC đồng quy tại điểm I.

Qua C ta kẻ đường thẳng song song với AB cắt đường MP tại W.

Dễ dàng chứng minh được tam giác CPW cân tại C nên CP=CW.

Từ đó theo định lí Thales : \dfrac{IA}{IC}=\dfrac{MA}{CW}=\dfrac{MA}{CP}=\dfrac{MA}{CN}

Theo định lí Menelaus cho tam giác ABC với sự thẳng hàng của K,M,N ta có : \dfrac{KA}{KC}.\dfrac{NC}{NB}.\dfrac{MB}{MA}=1\Rightarrow \dfrac{KA}{KC}=\dfrac{MA}{CN}

Vậy suy ra \dfrac{KA}{KC}=\dfrac{IA}{IC}\Rightarrow \dfrac{\overline{KA}}{\overline{KC}}=\dfrac{-\overline{IA}}{\overline{IC}}\Rightarrow (KIAC)=-1\;\;\;(3) (chú ý rằng K nằm ngoài và Inằm trong đoạn AC)

Từ (1)(2)(3) thì bổ đề chứng minh hoàn tất.

Trở lại bài toán : 

ddddd

Gọi M,N,P,Q lần lượt là các tiếp điểm trên AB,BC,CD,DA của (O).

Gọi X là giao điểm của PQ,MN. Gọi J là giao điểm của AC,BD.

Theo bổ đề thì OX \perp BD mà OH \perp BD nên O,H,X thẳng hàng.

Cũng theo bổ đề ta có (ACJX)=-1 kéo theo H(ACJX)=-1. Nhưng vì OH \perp BD\Rightarrow HJ \perp HX.

Theo định lí về chùm điều hòa ta có HJ là phân giác của góc AHC. Từ đó dễ dàng thấy được điều cần chứng minh \widehat{AHE}=\widehat{CHF}

Bài toán : Từ một điểm A nằm ngoài đường tròn tâm O, kẻ tiếp tuyến AB và cát tuyến AIK đến (O)với B là tiếp điểm, I nằm giữa A và K. Đường thẳng qua K vuông góc với OA cắt tia AB,IB lần lượt tại C,E. Chứng minh rằng C là trung điểm của AE.

Lời giải :

untitled

Vẽ tiếp tuyến AD với đường tròn với D là tiếp điểm và D khác B. Gọi J là giao điểm của BD với AK.

Dễ thấy rằng BD\parallel KE (cùng vuông góc với OA), khi đó theo định lí Thales trong tam giác AKC :

\dfrac{\overline{JB}}{\overline{KC}}=\dfrac{\overline{AJ}}{\overline{AK}}\Rightarrow \overline{KC}=\dfrac{\overline{JB}.\overline{AK}}{\overline{AJ}}

Tương tự, theo định lí Thales trong tam giác IKE : \dfrac{\overline{JB}}{\overline{KE}}=\dfrac{\overline{IJ}}{\overline{IK}}\Rightarrow \overline{KE}=\dfrac{\overline{JB}.\overline{IK}}{\overline{IJ}}

Mặt khác ta có (AJIK)=-1 (hàng điều hòa về đường tròn)

Suy ra

(JKAI)=(AIJK)=1-(AJIK)=2\Rightarrow \dfrac{\overline{AJ}}{\overline{AK}}:\dfrac{\overline{IJ}}{\overline{IK}}=2\Rightarrow \dfrac{\overline{IK}}{\overline{IJ}}=2\dfrac{\overline{AK}}{\overline{AJ}}\Rightarrow \dfrac{\overline{IK}}{\overline{IJ}}.\overline{JB}=2.\dfrac{\overline{AK}}{\overline{AJ}}.\overline{JB}\Rightarrow \overline{KE}=2\overline{KC}.

Điều này chứng tỏ C là trung điểm của KE.

Bài toán : Cho tam giác ABC, ba đường cao AD,BE,CF đồng quy tại H. Gọi I,K lần lượt là chân đường vuông góc hạ từ D,A xuống EF. Chứng minh rằng KH đi qua trung điểm M của ID.

Lời giải :

Banve

Gọi N là giao điểm của AD và EF

Do các tứ giác FECB và FHDB nội tiếp nên ta có \widehat{EFC}=\widehat{EBC}=\widehat{DFC}.

Nên FH là phân giác trong của tam giác FND, mà FH\perp AB nên FA là phân giác ngoài của tam giác FND.

Từ đó (AHND)=-1 (hàng điều hòa tia phân giác)

\Rightarrow K(AHND)=-1. Lại có KA\parallel ID (cùng vuông góc với EF).

Do đó theo định lí về chùm điều hòa ta có KH đi qua trung điểm M của ID.

Bài toán : Cho tam giác ABC có I,J lần lượt là tâm đường tròn nội tiếp và bàng tiếp góc A. Qua I,J lần lượt kẻ các đường thẳng DE,FG song song với BC với D,F thuộc đường thẳng AB và E,G thuộc đường thẳng AC. Chứng minh rằng : \dfrac{1}{\overline{DE}}+\dfrac{1}{\overline{FC}}=\dfrac{2}{\overline{BC}}

Lời giải :

Banve

Gọi K là giao điểm của AJ với BC

Ta sẽ chứng minh rằng (ABDF)=-1.

Thật vậy, ta có (AKIJ)=-1 (hàng điều hòa tia phân giác)

Do đó \dfrac{\overline{IA}}{\overline{IK}}=-\dfrac{\overline{JA}}{\overline{JK}}

Mà theo định lí Thales : \dfrac{\overline{IA}}{\overline{IK}}=\dfrac{\overline{DA}}{\overline{\overline{DB}}},\dfrac{\overline{JA}}{\overline{JK}}=\dfrac{\overline{FA}}{\overline{FB}}\Rightarrow \dfrac{\overline{DA}}{\overline{DB}}=-\dfrac{\overline{FA}}{\overline{FB}}\Rightarrow (ABDF)=-1

Theo hệ thức Decartes, ta có :

\dfrac{2}{\overline{AB}}=\dfrac{1}{\overline{AD}}+\dfrac{1}{\overline{AF}}\Leftrightarrow \dfrac{\overline{AB}}{\overline{AD}}+\dfrac{\overline{AB}}{\overline{AF}}=2

Mà cũng theo định lí Thales : \dfrac{\overline{AB}}{\overline{AD}}=\dfrac{\overline{BC}}{\overline{DE}},\dfrac{\overline{AB}}{\overline{AF}}=\dfrac{\overline{BC}}{\overline{FC}}\Rightarrow \dfrac{\overline{BC}}{\overline{DE}}+\dfrac{\overline{BC}}{\overline{FG}}=2\Rightarrow \dfrac{1}{\overline{DE}}+\dfrac{1}{\overline{FG}}=\dfrac{2}{\overline{BC}}

Đây là điều phải chứng minh.

Chia sẻ:

  • X
  • Facebook
Thích Đang tải...

Có liên quan

Từ khóa » đường Thẳng Gauss Newton